LSAT and Law School Admissions Forum

Get expert LSAT preparation and law school admissions advice from PowerScore Test Preparation.

 lsat_novice
  • Posts: 29
  • Joined: May 29, 2018
|
#46622
I thought that E was correct. I thought, "just because recent heavy rainfall filled the reservoirs doesn't mean the reservoirs will be full in the summer...it could be winter right now for all I know... and in the summer there may not be enough rainfall to avoid water rationing."

I don't understand why C is correct... what does the water company's ability to pump water have to do with water rationing? I think of water rationing as a measure set by the government... not a response to a water company that simply can't pump water fast enough.

Help please! :)
 Adam Tyson
PowerScore Staff
  • PowerScore Staff
  • Posts: 5153
  • Joined: Apr 14, 2011
|
#46639
Answer E certainly is attractive! The problem, though, is that it tells us what always happens, not anything special about this coming summer, like that it will be unusually dry. The author says the reservoirs are full, so we won't need to ration. We need an answer that says we still might need to ration, despite the full reservoirs. Answer E doesn't tell us much, if anything, about needing to ration, because it tells us what usually happens and our evidence is that we are currently unusually better prepared for that normal occurrence.

Answer C, though, adds new info that this summer we might be faced with an unusual challenge, which could offset our current good fortune. If there has been population growth, and we are having trouble getting water to all the customers, we might need to ration. In my view, pumping capacity isn't really the issue here - I think that part of the answer is more of a distraction than anything else. The issue is that while our supply is higher than normal, our demand is also higher than normal. While we might be able to avoid rationing, that increased demand makes me feel less certain, and that element of doubt it what weakens the argument.

In short, the argument is that supply is up, so we have enough. Answer E is that demand is also up. That weakens the claim that we now have enough. I hope that helps clarify it!
 lsat_novice
  • Posts: 29
  • Joined: May 29, 2018
|
#46640
Thanks that helps! But it's hard not to be distracting by the issue of pumping capacity.
 whardy21
  • Posts: 48
  • Joined: Sep 30, 2018
|
#65736
I was between B and C. Can you please explain to me why B is incorrect and why C is correct? Thank you.
 George George
PowerScore Staff
  • PowerScore Staff
  • Posts: 48
  • Joined: Jun 07, 2019
|
#65765
@whardy21

Sure! (B) is wrong because it's vague and too weak. First, the answer is vague. Just because the rain didn't reach the "deep underground water system" doesn't mean there's a problem with the supply down there to begin with! This is the major issue with answer (B). The first takeaway is to beware vague answer choices that don't give you enough information to work with - they leave you guessing about what the actual impact of that information would be on the author's argument. Second, even if you assumed that there was a deficiency deep underground, just because a "small part" of the water supply comes from something else doesn't mean the city will have to do water rationing! After all, they just filled the reservoirs - so the needed water could come from there. The second takeaway is that answers that are too weak can actually be wrong on Weakener Qs!
User avatar
 CristinaCP
  • Posts: 28
  • Joined: Sep 17, 2023
|
#104355
Hi Powerscore,

I got this question right, but I found it difficult to confidently eliminate [A]. I ultimately eliminated [A] because it just says that there has been rationing in the last three years. I figured that just because it happened in the last three years doesn't mean it has to happen this year. After all, there was more rain, so maybe this is the year where rations are unnecessary. For that reason, I thought A could be true without weakening the argument.

Is this the right reasoning for why A is wrong?
User avatar
 Jeff Wren
PowerScore Staff
  • PowerScore Staff
  • Posts: 389
  • Joined: Oct 19, 2022
|
#104381
Hi Cristina,

Yes, your reasoning is correct.

One thing to notice in the argument are the words "recent unexpectedly heavy rainfalls" (my emphasis). The fact that the heavy rainfalls were unexpected suggests that there was more rain than is typical. Given this fact, even if the typical amount of rainfall in the past few years required rationing, this unusually heavy amount may mean that rationing will not be needed this summer.

At its core, this argument is about supply and demand. The argument is basically, since the water supply has increased, therefore demand doesn't need to be restricted/limited (i.e. rationed).

Whenever you get an argument mentioning supply or demand, it's always a good idea to consider the other side of the equation. In other words, just because supply increased doesn't mean that there will be enough water if demand also increases.

Answer C addresses the increase in demand (due to increasing population) while also mentioning another restriction on the supply available. (Even if there is enough water in the reservoirs, it may not be possible to get the water to all of the customers in time without rationing due to the limits on the water pumps.)

Get the most out of your LSAT Prep Plus subscription.

Analyze and track your performance with our Testing and Analytics Package.